Wrong PMHNP Questions

¡Supera tus tareas y exámenes ahora con Quizwiz!

Alice is a 68-year-old woman who presents to you with concerns about her memory. She explains that her mother and grandmother both experienced dementia and she wants to do what she can to prevent this terrible disease. Alice is treated for hypertension, which is well controlled; other than that she is in good health. She is socially and physically active and participates in a monthly cooking class, volunteers at her church, and plays bridge twice a week at the senior center. She says, "I understand that I am losing brain cells at my age, but I would still like to keep my mind and body active." Which is the best response to Alice?

"Although most brain development occurs early in life, we still form some new brain cells in a couple of areas of the brain during adulthood. You should continue with your activities because they offer excellent physical and mental health benefits and are neuroprotective." While it was once thought that the brain neurons did not regenerate, we now know that while most brain development happens in early life, we continue to form some new brain cells throughout life. As we age, we need to engage in activities that keep our brains healthy by encouraging this growth. Examples are diet, exercise, socialization, and cognitive stimulation.

A mobile crisis team responded to an emergency call by a residential care home (RCH) manager. A 65 year old resident had become increasingly agitated, insisted that the RCH cook was poisoning her food, and verbally threatened the cook. The RCH manager had found several days of the residents medication hidden in her dresser. Which is the most appropriate intervention by the mobile crisis team?

* Admission to the crisis stabilization unit* The universal bill of rights for mental health patients, restate of bill of rights for mental patients established by mental health systems act of 1980 supports evaluation and treatment in the least restrictive environment. The crisis stabilization unit represents least restrictive environment.

a pt was started on antipsychotic med 2 weeks ago and is now experiencing the following symptoms: inability to sit still, pacing, feelings of inner restlessness . He does not complaint of any anxiety or being worried about anything in particular. Which of the following rating scales would be appropriate?

* Barnes Akathisia Rating Scale (BARS)* The pt is experiencing akathisia. The BARS can be very useful in identifying akithesia, assess severity, and monitoring changes in symptoms.

The PMHNP asks the patient to do serial 7s, what aspect of the MSE would the NP be assessing ?

* Concentration* Subtracting serial 7's from 100 is a simple tasks that requires intact concentration and cognitive abilities.

Your pt suffers from insomnia and bipolar DO. She tells you that she likes to take her Lurasidone 20mg (Latuda) at bed because it is easier for her to remember. She does not like to gain weight so she eats two 90-calorie packages of harm slices with the Luradisone before she goes to bed. She complaints of not being able to go to sleep very quickly? You respond?

* I recommend that you take in 360 calories of a carbohydrate at bed time to induce sleepiness* You must eat 360 calories of food to get proper absorption of Lurasidone. If you do not, you only get 1.2 of the medication absorbed into the body. In addition, protein tends to awake people while carbohydrates tends to make them sleepier. Turkey is the exception d/t tryptophan.

Which of the following illustrates the role of the psych NP in reducing the stigma of mental illness through communication education and primary prevention.

* Professional speaker interviewed by the morning television talk show on depression* Primary prevention through community education is best illustrated by reaching the broadest general audience with info about depression, via television, radio, newspaper.

Dementia screening for the general population is not recommended for the following reasons?

* The service has unknown balance of benefit and harm: current treatments have limited effectiveness in modifying instrumental activities of daily living * Dementia screening is not indicated for general population because the target condition is largely prevalent in the elderly. The balance of benefit and harm is unknown and current treatments have limited effectiveness in modifying instrumental activities of daily living.

Which of the following findings would raise concern in an annual exam of a 76 year old woman?

*AST 85 & ALT 45* *The AST is elevated, normal range is 5-40, and ALT is normal here, range is 7-56. When AST > ALT one suspects chronic liver disease or recent MI. Further assessment is warranted including ruling out alcohol dependence.

In a hospital psychiatric consultation setting, what is one of the most common mental illness diagnoses?

*Adjustment Disorder* Adjustment disorders are very common. They are one of the most common diagnoses given in a hospital psychiatric consultation setting, frequently reaching up to 50 percent. About 5 to 20 percent of patients in an outpatient mental health clinic meet the criteria for this diagnosis as well.

A pt comes into office in a full manic episode. this pt has dx of bipolar and alcohol use disorder. You have a report from the pts chemical dependency counselor indiciating that inpatient chemical dependency counseling is needed. your patient has been on the fence about inpatient chemical dependency treatment in the past. You assess the pt and find a strong odor of alcohol and reporting visual hallucinations, flight of ideas, lack of sleep of 3 days, and suicidal ideation and plans to drive to another state to game. The pts brother corroborates info. What do you do?

*Admit him to the local psychiatric unit, with an order for the collection of lab values.* While he is both manic and a danger to himself, he should be hospitalized. The patient will not be able to participate in any programming until he is out of his manic episode. Going home is not an option unless the pt has somone who can be with him 24/7 to monitor for withdrawal from alcohol, his manic symptoms and suicidal ideation.

The NP noticed that NP profession is labelled as paraprofessional in a flyer. In the meeting the NP asks the organization responsible for the pamphlet why they have NPs listed as paraprofessionals, when by law NPs provided services within the same capacity as MDs. The NP goes on to explain what an NP does and the various areas of practice. What is this?

*Advocating for the role and value of the NP*

The PMHNP evaluates a 3-year-old male and makes the dx of ADHD. Following FDA labeling of approved meds for children, which medication can be started at 3 years of age?

*Amphetamines* Only amphetamines are approved in children ages 3 and older. Methylphenidate and guanfacine are approved in ages 6 and older. Fluoxetine and bupropion are not approved for ages under 8.

Which of the following lifestyle factors poses the greatest risk for depression, irritability, liver problems, and hypertension in a college student?

*Anabolic steroid use* Anabolic steroid use poses the greatest risk of the four lifestyle drug use patterns. Anabolic steroid use is associated with increased irritability, aggression, euphoria, increased energy, sexual arousal, mood swings, distractibility, forgetfulness, confusion. With time, anabolic steroid use is associated with increased risk of heart attacks, strokes, blood clotting, cholesterol changes, hypertension, depression mood, fatigue, restlessness, loss of appetite, insomnia, reduced libido, muscle and joint pain, and severe liver problems, including hepatic cancer, Males can have reduced sperm production, shrinking of the testes, and difficulty or pain urinating. There can be breast enlargement in men and masculinization of women's bodies. Both sexes can experience hair loss and acne Intravenous or IM use of the drug and needle sharing puts uses at risk for HIV and Hep A and C as well as endocarditis.

What is the most common sexual dysfunction side effect reported by women who are prescribed a selective serotonin reuptake inhibitor (SSRI)?

*Anorgasmia* Anorgasmia is the most commonly reported sexual dysfunction side effect in women who are taking SSRIs. This is defined as the persistent inability to achieve orgasm despite responding to sexual stimulation.

In co-leading a group of elementary children of divorcing parents, a child shared that his parents were arguing over the weekend and calling each other names. What is the most appropriate intervention at this juncture?

*Ask the child how he was affected by the incident and ask other children how they handle similar situations* Self-disclosure: Use judgement as to the appropriateness of letting a child go into detail about personal matter in a group. In an elementary school group, you may not want to let a child go into detail about a parental fight. However, facilitating sharing of similar feelings, thoughts and coping strategies by the other children can be one means to help the entire group to recognize that they are not responsible for their parents arguments, behaviors, thoughts or feelings.

Which of the following best depicts the PMHNP in a grassroots mental health advocacy role?

*Attending a town hall meeting to seek a timely police response to rising DV crisis call* The response involved a specific health issue with a proposed action to improve care is time to police response to domestic violence crisis calls.

In assessing patient risks for binge drinking, which of the following statatements is not correct?

*Binge drinking is higher in urban, densely, populated areas than rural sparsely populated areas.* There is little variation seen in binge and heavy alcohol use rates by population density, there are regional differences in alcohol use, but not specifically binge drinking rates, with highest alcohol use in Western States and lowest use in Southern States. The other ressponses are accurate regarding binge drinking.

A mom ask's the NP if her son's fixation on rules and getting distressed when things are out place in his room is normal. Understanding Piaget's stages of development, the NP is aware that her son is having difficulty mastering what cognitive developmental stage?

*Concrete operations* Children who have a hard time of with syllogistic reasoning, in the sage of concrete operations in ages 7-11 years, can be orver invested in rules and develop OCD-like behaviors.

Which of the following is not associated w/ increased validity in a clinical research trial?

*Confounder* A confounder is factor that distorts the true relationship of the study variable of interest by virtue of also being related to the outcome of interest. Confounders are often unequally distributed among the groups being compared.

During an outpatient medication eval a depressed client reports persistent anger toward his former boss after losing his job due to arguments and aggressive behavior toward coworkers. The client has been waiting outside the plant in the afternoon, watching for the boss to leave to confront him regarding his terminated employment. What is the responsibility of the NP?

*Consult state board of nursing regarding state law requirement on reporting potential harm* Tarasoff v Regent of Uni of California (1974) established that a mental health professional has a "duty to warn" potential victims of impending danger from a patient. State laws on duty to warn, vary by state. It is the professional responsibility to know what is required in the state that one practices in.

A 49-year-old female arrives in the emergency room. She has a long history of IV drug use. Over the past year, the patient's personality has changed, and she is more withdrawn and subdued. Her memory has deteriorated, and she has lost 45 pounds in the past five months. She complains of weakness and difficulty maintaining her balance. During your examination, she scores 21/30 on the mini-mental status exam.

*Correct answer*: Human immunodeficiency virus Human immunodeficiency virus (HIV) results in a wide spectrum of neuropsychiatric symptoms due to opportunistic infections, neoplasms, or direct infiltration of the central nervous system. HIV-associated dementia presents with motor abnormalities, cognitive impairment and decline, apathy, and social withdrawal. Neurologic findings might include weakness, imbalance, or ataxia. This patient's history of IV drug use and recent rapid weight loss are also indicative of HIV.

What percent of the incarcerated population in the United States has a severe mental illness?

*Correct answer: 15% to 24%* Studies estimate that 15% to 24% of the incarcerated population in the United States has a severe mental illness. Unfortunately, a lack of synchronized care in the critical justice system results in repeated incarcerations and fragmented psychiatric care for this population.

By the time they turn 18 years old, what percent of adolescents regularly smoke tobacco?

*Correct answer: 20%* Most adolescents in the United States experiment with tobacco use. By the age of 18, at least 20% of individuals smoke tobacco regularly. More than 80% of these people attempt to quit, but only 5% will be successful. Daily smoking in the United States used to be very rare; however, it has become more prevalent in the past decade, especially among teenagers.

What happens at four weeks in development.

*Correct answer: 4 weeks* At four weeks, landmarks of normal behavioral development include Hands fisted Head sags but can hold head erect for a few seconds Follows moving objects to midline Responds to speech Smiles preferentially at mother

In the United States, what percent of the homeless population has co-occurring substance use disorders and serious mental illness?

*Correct answer: 50%* Approximately 50% of the homeless population in the United States has a substance use disorder co-occurring with serious mental illness. Schizophrenia occurs in 15% to 45% of homeless persons in the United States.

What percent of United States citizens have private insurance?

*Correct answer: 64%* Health insurance coverage in the United States is as follows: *64%* have private insurance *16.3%* are uninsured *14.5%* have Medicare *15.9%* have Medicaid

According to the United States Centers for Disease Control and Prevention, who does not need to receive the annual influenza vaccination?

*Correct answer: A healthy 25-year-old male* Healthy individuals between the ages of 19 and 49 are not required to receive the annual influenza vaccine unless they meet any of the criteria below. The following people should always receive the influenza vaccine: Children from six months to 18 years of age Anyone aged 50 years and older Anyone who lives in a nursing home or in a long-term care or assisted living facility Anyone with a chronic health condition including asthma, diabetes, renal or hepatic dysfunction, HIV, and neuromuscular disorders Pregnant women Health care personnel Caregivers of children younger than five years

Which statement accurately defines an unspecified intellectual disability?

*Correct answer: A term used when a child is five years old or older and cannot be reliably assessed due to a physical or mental impairment.* Global developmental delay describes a child under the age of five who seems to be falling behind developmentally, but the practitioner cannot reliably assess the degree of impairment. Borderline intellectual functioning describes a person nominally ranked in the IQ rank of 71 to 84 who does not have the coping problems associated with an intellectual disability. Autism spectrum disorder forms in early childhood and results in impaired social interactions and communication. These patients usually demonstrate stereotyped behaviors and interests. Unspecified intellectual disability is a term used when a child is five years old or older and cannot be reliably assessed due to a physical or mental impairment.

Incidence of Autism Spectrum Disorder?

*Correct answer: About 1%* Autism spectrum disorder (ASD) affects about 1% of the population in the United States. Other important facts regarding ASD: ASD is more common in children with a family history of pervasive developmental disorders The concordant rate for an identical twin with autism is 60% The incidence is 2 to 5 cases per 10,000 in the US The male to female ratio is 4:1 Onset of symptoms is before age 3 About 10% of people with autism also have a genetic or chromosomal condition such as Down syndrome or fragile X syndrome

As a nurse practitioner, you are responsible for assessing cranial nerves as a part of the neurological exam. What cranial nerve assessment tests the strength of the sternocleidomastoid and trapezius muscles against the resistance of the provider's hands?

*Correct answer: Accessory spinal* The accessory spinal (cranial nerve XI) assessment tests the strength of the sternocleidomastoid and trapezius muscles against the resistance of the provider's hands. The trigeminal nerve (cranial nerve V, sensory division) assessment tests for tactile perception of the facial skin. The trigeminal nerve (cranial nerve V, motor division) assessment tests for masseter muscle strength. The facial nerve (cranial nerve VII, motor division) assessment checks for flaccid paralysis.

Which complementary and alternative medicine (CAM) technique is based on manipulating the flow of energy throughout the body?

*Correct answer: Acupuncture* Both acupuncture and acupressure are based on the idea from Chinese medicine that vital energy called chi flows along specific pathways in the body. Manipulating this energy using needles or pressure can correct imbalances. Acupuncture and acupressure are thought to produce their therapeutic effects by aiding in the activity of endorphins and the immune system. It is also thought that they alter brain chemistry by changing the release of certain neurohormones and neurotransmitters.

All of the following are examples of Irvin Yalom's curative factors except:

*Correct answer: Adjourning* Irvin Yalom was the first person to put a theoretical perspective on group work. He identified 10 curative factors that differentiate group therapy from individual therapy: Instillation of hope Universality Altruism Increased development of socialization skills Imitative behaviors Interpersonal learning Group cohesiveness Catharsis Existential factors Corrective refocusing Irvin Yalom also believed that all groups go through specific phases: Pre-group Forming Storming Norming Performing Adjourning

Why should we administer thiamine to alcohol use disorder client who has only been using alcohol and had possible caloric deficit?

*Correct answer: Administer thiamine* Individuals with chronic alcohol misuse often consume most of their calories from alcohol and are at high risk for thiamine deficiency. *A thiamine-deficient person can develop Wernicke encephalopathy if they attempt to metabolize food.* You must administer thiamine immediately for this patient. The other choices are also important but do not require immediate action.

Which circadian rhythm sleep-wake disorder subtype is characterized by falling asleep and waking earlier than desired?

*Correct answer: Advanced sleep phase type* Circadian rhythm sleep-wake disorders are characterized by a mismatch between a person's biological clock and the environment. There are five subtypes: *Delayed sleep phase type:* falling asleep and waking later than desired *Advanced sleep phase type*: falling asleep and waking earlier than desired *Irregular sleep-wake type*: falling asleep and waking at random times *Non-24-hour sleep-wake type*: falling asleep and usually waking progressively later than desired *Shift work type*: sleepiness associated with changes in work schedule Jet lag refers to feeling sleepy or "hungover" after crossing time zones. It is no longer considered a sleep disorder.

An 82-year-old woman arrives at your office accompanied by her son. He explains that over the past year his mother has had trouble remembering once-familiar things like telephone numbers and friends' names. She also does not recognize family members who visit often. She has become unusually moody and struggles with written tasks. Based on this information, what is the most likely diagnosis?

*Correct answer: Alzheimer's disease* It is often difficult to differentiate between the types of dementia. This patient's presentation most likely aligns with Alzheimer's disease due to her deficits in language, motor function, and recognition. *Dementia with Lewy bodies* is usually accompanied by extrapyramidal symptoms. *Huntington's disease* is accompanied by spasmodic movement and incoordination. Patients with *frontotemporal dementia* are usually extremely talkative and disinhibited.

The Health Information Technology for Economic and Clinical Health (HITECH) Act was included under which 2009 law?

*Correct answer: American Recovery and Reinvestment Act* In February 2009, President Barack Obama signed into law the American Recovery and Reinvestment Act (ARRA) (Pub. L. 111-5). As a response to the Great Recession, this law aimed to save and create jobs while financing infrastructure, education, health care, and renewable resources. The ARRA includes the Health Information Technology for Economic and Clinical Health (HITECH) Act, which attempts to update the American infrastructure, including the use of electronic health records.

A patient presents with psychosis, dry mouth, hyperpyrexia, mydriasis, restlessness, and tachycardia. What possible psychiatric emergency would you suspect?

*Correct answer: Anticholinergic intoxication* Psychosis, dry mouth, hyperpyrexia, mydriasis, restlessness, and tachycardia are symptoms of anticholinergic intoxication. The nurse practitioner should discontinue the offending medication or substance and consider IV physostigmine and benzodiazepines. Remember, antipsychotics are contraindicated in this situation. Alcohol withdrawal would present with irritability, nausea, vomiting, insomnia, malaise, autonomic hyperactivity, and shakiness. Benzodiazepine intoxication would present with sedation, somnolence, and ataxia. Catatonic schizophrenia would present with marked psychomotor disturbance (either excitement or stupor) and exhaustion.

Who should not get shingles vaccine?

*Correct answer: Anyone over the age of 50* Shingles, also known as herpes zoster, is an inflammatory condition where a virus produces painful vesicular eruptions along the nerves from one or more dorsal root ganglia. The shingles vaccine is recommended for anyone older than 50 whether or not they recall having had chickenpox, which is caused by the same virus as shingles. In May 2006, the Food and Drug Administration (FDA) approved a VZV vaccine (Zostavax) for use in people 60 and older who have had chickenpox. In March 2011, the FDA extended the approval to include adults ages 50-59. A new shingles vaccine called Shingrix was licensed by the FDA in 2017 for adults age 50 and older. The shingles vaccine should not be given to: People with a weakened immune system People with HIV, AIDS, or a T-cell count below 200 Patients being treated with high-dose steroids

Melatonin is known to interact with which substance?

*Correct answer: Aspirin* Melatonin is known to interact with aspirin, NSAIDs, beta blockers, corticosteroids, valerian, kava kava, and alcohol.

Lifetime risk of suicide for bipolar disorder? Completed suicide?

*Correct answer: Bipolar disorder* Bipolar disorder accounts for 25 percent of all completed suicides. The lifetime risk of suicide in those diagnosed with bipolar disorder is 15 times greater than that of the general population. These individuals are more likely to complete suicide if they have a history of suicide attempts and have spent the majority of the past year in a depressed episode.

The cerebrum contains all of the following areas of the brain except:

*Correct answer: Brainstem* The brain itself is divided into two distinct anatomical regions: the cerebrum and the brainstem. The cerebrum contains the cerebral cortex, limbic system, thalamus, hypothalamus, and basal ganglia. The brainstem includes the midbrain, pons, cerebellum, medulla, and reticular formation.

Tanner Stages define physical measurement in the development of primary and secondary sex characteristics in both females and males. What is stage 2 of a female's breast development?

*Correct answer: Breast bud stage with elevation of breast and papilla* Tanner Stages define physical measurement in the development of primary and secondary sex characteristics in both females and males. Tanner Stages of a girl's breast development are as follows: Stage 1: Prepubertal Stage 2: Breast bud stage with elevation of breast and papilla Stage 3: Further enlargement of breast and areola Stage 4: Areola and papilla form a secondary mound above the level of the breast Stage 5: Projection of papilla related to recession of areola

A 63-year-old female with a long history of generalized anxiety disorder tells you she was out shopping two days ago when she suddenly felt dizzy, experienced heart palpitations and pressure on her chest, and felt a sense of impending doom. Shortly after, she "passed out" and woke up minutes later with a crowd of people surrounding her. She states that she then felt better and decided to keep shopping. What is the most likely diagnosis?

*Correct answer: Cardiovascular disease* Regardless of this patient's psychiatric history, organic causes of her symptoms must be ruled out. This patient should be referred to a cardiac specialist or her primary care provider for further evaluation. Anxiety disorders such as panic disorder and social anxiety very rarely cause unconsciousness.

Use of herbal substances in treating mental disorders is becoming more and more popular. Which herbal substance is used in treatment of insomnia?

*Correct answer: Catnip* Common herbals with psychoactive effects used to treat insomnia: Valerian Catnip Chamomile, which also helps with anxiety Other herbals with psychoactive effects: *Ginkgo* which helps with delirium, dementia, and sexual dysfunction *Black cohosh*, which is used to treat menopausal symptoms, premenstrual syndrome, and dysmenorrhea *Belladonna*, which is used to treat anxiety *Ginseng*, which helps with depression and fatigue

Which statement is true regarding bipolar disorder and pregnancy? Lithium (Lithane, Lithobid) is safe during the first and third trimesters Carbamazepine (Tegretol, Carbatrol, Equetro) is a safe alternative to lithium carbonate Clonazepam (Klonopin) does not carry an increased risk of major fetal malformations Divalproex sodium (Depakote) is a safe alternative to lithium carbonate

*Correct answer: Clonazepam (Klonopin) does not carry an increased risk of major fetal malformations Commonly used mood-stabilizing medications for bipolar disorder carry an increased risk of major fetal malformations. However, clonazepam, lorazepam, and alprazolam are effective in the management of acute manic episodes as an adjuvant to maintenance therapy in lieu of antipsychotics. As an adjuvant to lithium or lamotrigine, clonazepam may result in increased time between cycles and fewer depressive episodes.* During the first trimester, the use of lithium (Lithane, Lithobid) increases the risk of fetal cardiac malformations to 7.7%. Lithium can be used in the second and third trimesters, but it must be stopped peripartum due to the rapid fluid shifts during birth. Both carbamazepine (Tegretol, Carbatrol, Equetro) and divalproex sodium (Depakote) increase the risk of neural tube defects.

Systemic effects of hyponatremia include all of the following except:

*Correct answer: Convulsions* Other presentations of *hyponatremia* include the following: Apprehension Seizures Coma Hypotension Tachycardia Decreased urine output Weight gain Edema Ascites Jugular vein distension Presentations of *hypernatremia:* Convulsions Pulmonary Edema Thirst Fever Dry mucous membranes Hypotension Tachycardia Low jugular venous pressure Restlessness

According to Irvin Yalom, which curative factor occurs when participants reexperience family conflicts within group therapy?

*Correct answer: Corrective refocusing* Irvin Yalom was the first person to put a theoretical perspective on group work. He identified 10 curative factors that differentiate group therapy from individual therapy: *Instillation of hope Universality: Universality occurs as participants discover that others have similar problems, thoughts, or feelings. Altruism: Altruism results from sharing oneself with another. Increased development of socialization skills Imitative behaviors Interpersonal learning: Interpersonal learning allows participants to increase their adaptive interpersonal relationships through group therapy sessions. Group cohesiveness Catharsis Existential factors Corrective refocusing* *Corrective refocusing occurs when participants reexperience family conflicts in the group. These experiences allow them to recognize and change their problematic behaviors.*

Alcohol dependence has increased or decreased triglycerides?

*Correct answer: Decreased triglycerides In alcohol dependence, triglycerides are usually elevated.* Alcohol dependence and abuse produce characteristic laboratory findings, including the following: Elevated mean corpuscular volume (MCV) Increased prothrombin time Aspartate aminotransferase (AST) elevation greater than 40% Alanine aminotransferase (ALT) elevation greater than 20% Elevated amylase Low white blood cell count Elevated uric acid Elevated triglycerides

According to Erik Erikson, during which psychosocial stage would you expect a person to develop a sense of unity in life's accomplishments?

*Correct answer: Ego Integrity vs. Despair* Ego Integrity vs. Despair is the final stage of psychosocial development. Successful resolution of this stage is characterized by a sense of unity in life's accomplishments. Failure to resolve this stage leads to regret over lost opportunities of life. Erik Erikson's stages of psychosocial development: *Trust vs. Mistrust*: birth to 12-18 months *Autonomy vs. Shame and Doubt*: 12-18 months to 3 years *Initiative vs. Guilt:* 3 to 5-6 years *Industry vs. Inferiority*: 5-6 years to adolescence *Identity vs. Role Confusion*: adolescence to adulthood *Intimacy vs. Isolation*: adulthood *Generativity vs. Stagnation:* middle adulthood *Ego Integrity vs. Despair*: late adulthood

Risk factors for developing posttraumatic stress disorder (PTSD) are divided into three categories: pretraumatic, peritraumatic, and posttraumatic. Which of the following is a pretraumatic risk factor for developing PTSD?

*Correct answer: Emotional disturbances prior to age six* Risk factors for developing posttraumatic stress disorder (PTSD) are divided into three categories: pretraumatic, peritraumatic, and posttraumatic. *Pretraumatic risk factors:* Female gender Emotional disturbances prior to age six Childhood mental illness Externalizing behaviors Lower socioeconomic status Low education Childhood adversity Fatalistic or self-blaming coping mechanisms Minority social/ethnic status Low intelligence Poor social support Family history of psychiatric disorders *Peritraumatic risk factors:* Severity of trauma Duration of trauma Perceived life threat Personal injury Trauma perpetrated by a loved one or caregiver Witnessing a threat directed toward a loved one or caregiver Dissociation during trauma For military personnel, witnessing atrocities or killing the enemy *Posttraumatic risk factors:* Negative appraisals Inappropriate coping strategies Development of an acute stress reaction Subsequent exposures to reminders Subsequent adverse life events and financial losses Lack of social support

What term is defined as tension between what one wants to do and what is morally right?

*Correct answer: Ethical conflict* Ethical conflict is defined as the tension between what one wants to do and what is morally right. Ethical dilemmas are conflicts between different ethical perspectives. Ethics is the branch of knowledge that deals with moral principles. Professional ethics refers to the branch of knowledge that deals with moral behavior when in a professional role.

For an act to be considered criminal, it must have two components: actus reus and mens rea. What is mens rea?

*Correct answer: Evil intent* For an act to be considered criminal, it must have two components: actus reus and mens rea. Actus reus refers to voluntary conduct. Voluntary conduct is deemed impossible if the offender's mental status is deficient, abnormal, or diseased in a way that inhibits rational intent. Mens rea refers to evil intent. Evil intent is defined as the resolve to do harm. Neither behavior nor intent alone is enough to convict a person of a crime.

According to Irvin Yalom, what would you expect to occur during the forming phase of group therapy?

*Correct answer: Expectations are identified and boundaries are established* Irvin Yalom was the first person to put a theoretical perspective on group work. He believed that all groups go through specific phases: Pre-group Forming Storming Norming Performing Adjourning During the forming phase: Members appear concerned about self-disclosure and rejection Goals and expectations are identified Boundaries between the leader and participants are established Trust and rapport begin to develop

Belladonna is a common non-psychoactive herbal supplement used to cure illness and maintain health.

*Correct answer: False* Belladonna is a psychoactive herbal supplement that produces psychological effects believed to aid in relieving psychiatric symptoms. Practice of herbal medicine originated in China and is the oldest system of medicine. This method of healing relies on plants to cure illnesses and maintain health. Similar to prescription medications, many plants contain active compounds that produce physiological and psychological effects. Food and Drug Administration approval is not required; thus, no uniform standards ensure quality control or potency.

The four major dimensions of recovery include health, home, continuity, and community.

*Correct answer: False* The four major dimensions of recovery include health, home, purpose, and community.

Vitamin E, when paired with warfarin, enhances the healing process of clotting disorders and is often prescribed in conjunction with anticoagulant therapy.

*Correct answer: False* Vitamin E interacts with warfarin by increasing anticoagulant effects of antiplatelet drugs and statins, increasing additive effects and the risk of rhabdomyolysis (muscle wasting).

When initiating treatment with clients who have newly diagnosed bipolar disorder, clients should be seen monthly for titration of medications and monitoring of serum blood levels. True or false?

*Correct answer: False* When initiating treatment with clients who have newly diagnosed bipolar disorder, clients should be seen weekly, not monthly, for titration of medications and monitoring of serum blood levels.

Each of the following is a culture-related term for describing panic attacks except:

*Correct answer: Fearful spells* Cultures across the world describe panic attacks using different phrases or terms: *Vietnamese*: "Hit by the wind" *Latin Americans*: "Attack of the nerves" *Cambodians*: "Soul loss" "Fearful spells" refers to a limited symptom attack that does not meet the full criteria for a panic attack.

All of the following are true regarding females with bipolar disorder except:

*Correct answer: Females are more likely to develop psychotic features* Gender does not appear to be related to the likelihood that someone with a bipolar disorder will develop psychotic features. Females with bipolar disorder are more likely to experience rapid cycling and mixed states and are more likely to experience depressive episodes. They also have higher rates of comorbid alcohol use disorders and eating disorders.

Each of the following statements about the development and course of obsessive-compulsive disorder is true except

*Correct answer: Females have an earlier age of onset than males* In obsessive-compulsive disorder (OCD), males have an earlier age of onset. Up to 25 percent of males with OCD are diagnosed before they turn 10 years old. Onset in childhood often indicates that it will be a lifelong illness. For OCD, the mean age of onset is 19.5 years, and onset after the age of 35 is very rare. Overall, females are affected at a slightly higher rate than males. The 12-month prevalence of OCD in the United States is 1.2%.

Which of the following is a common non-psychoactive supplement used to help cure symptoms of illness and maintain health?

*Correct answer: Fish oil* Fish oil is a common non-psychoactive supplement used to cure symptoms of illness and maintain health. Other common non-psychoactive dietary supplements with physiological (not psychological) effects used to cure illnesses and maintain health are as follows: Omega-3 fatty acids Tryptophan Vitamin E Melatonin SAM-e St.-John's-wort is a psychoactive herbal supplement. Magnesium is an electrolyte supplement. Belladonna is a psychoactive herbal supplement.

Competence to be executed was established in what Supreme Court case?

*Correct answer: Ford vs. Wainwright* Competency to be executed was established in the Supreme Court case of Ford vs. Wainwright. The requirements to establish the competence to be executed are as follows: The person must be aware of the retributive element of punishment. The person must be in the best position to make whatever peace is appropriate with his or her religion. Someone who is determined competent to be executed preserves the right to recall a forgotten detail of the crime that may prove exonerating. Most national medical organizations agree that it is unethical for any clinician to participate in state-mandated executions. Rather, nurse practitioners have a duty to preserve life, and this transcends any other competing requirements. The 1976 case of O'Connor vs. Donaldson ruled that harmless mentally ill patients cannot be confined against their will if they can survive outside. This case determined that the presence of a mental illness alone cannot justify involuntary hospitalization. In 1979, Rennie vs. Klein determined that patients have the right to refuse any treatment and use an appeal process. Durham vs. United States determined that an individual is not criminally responsible if the unlawful act was the product of mental illness. This case is known for originating the insanity defense.

You are the nurse practitioner treating a 19-year-old female. During your initial assessment, the client complains of not sleeping well. You understand that disruptions in sleep patterns have etiology in all of the following except:

*Correct answer: Genetics* Genetics have not been substantiated as being an etiological cause of insomnia. The etiology of insomnia is found in the following: Dysfunction in sleep-wake circuits of the brainstem Neurochemical imbalances impinging on these circuits May be stress-related in brief episodic insomnia

In common psychiatric disorders, neurotransmitters are implicated in the complex pathophysiology of certain disease processes. It is important to understand neurotransmitters in the central nervous system in order to better diagnose and treat common psychiatric illnesses. Which neurotransmitter's general function involves memory and sustained autonomic functions?

*Correct answer: Glutamate* The general function of glutamate involves memory and sustained autonomic functions. The general function of dopamine involves normal functioning of thinking, decision-making, reward-seeking behavior, fine muscle action, and integrated cognition. The general function of norepinephrine involves alertness, orientation, fight-or-flight, learning, memory, and focus and attention. The general function of serotonin involves regulation of sleep, pain perception, mood states, temperature, aggression, and libido.

What is the most commonly used clinician-administered anxiety rating scale?

*Correct answer: Hamilton Anxiety Scale* The Hamilton Anxiety Scale (HAM-A) is the most commonly used clinician-administered anxiety rating scale. It is best used in the evaluation of anxiety severity and tracking the efficacy of anxiety treatments over time. The HAM-A is based on various domains of anxiety including anxious mood, fears, sleep disturbance, somatic complaints, tension, and observed behavior. The severity of each domain is ranked from 0 (not present) to 4 (severe). Scoring is as follows: Score of 14-17: Mild anxiety Score of 18-24: Moderate anxiety Score of 25-30: Severe anxiety

Health Information Technology for Economic and Clinical Health Act?

*Correct answer: Health Information Technology for Economic and Clinical Health Act* In February 2009, President Barack Obama signed into law the American Recovery and Reinvestment Act (ARRA) (Pub. L. 111-5). As a response to the Great Recession, this law aimed to save and create jobs while financing infrastructure, education, healthcare, and renewable resources. The ARRA includes the Health Information Technology for Economic and Clinical Health (HITECH) Act, which attempts to update the American infrastructure, including the use of electronic health records.

What term best describes the organizational and institutional structures through which an economy makes choices regarding the production, consumption, and distribution of medical services?

*Correct answer: Health care system* *Health care system* describes the organizational and institutional structures through which an economy makes choices regarding the production, consumption, and distribution of medical services. *Health production function* is a mathematical expression that shows the relationship between an individual's health and a number of other variables, including the amount of health care consumed. A *health alliance* is a public agency that uses its bargaining power to negotiate competitive prices for health insurance from the private insurance market. *Health economics* is related to the value, effectiveness, and efficiency of care received.

What term best describes the relationship between an individual's health and a number of other variables, including the amount of health care consumed?

*Correct answer: Health production function* Health production function is a mathematical expression that shows the relationship between an individual's health and a number of other variables, including the amount of health care consumed. Health care system describes the organizational and institutional structures through which an economy makes choices regarding the production, consumption, and distribution of medical services. Health economics is related to the value, effectiveness, and efficiency of care received. Health alliance is a public agency that uses its bargaining power to negotiate competitive prices for health insurance from the private insurance market.

What findings are important to collect in anorexia?

*Correct answer: Height and weight* This patient presents with symptoms indicative of anorexia nervosa. Height and weight will help you establish the severity of her diagnosis. Individuals with anorexia nervosa achieve or maintain a significantly low weight. Severity of anorexia nervosa is characterized as mild, moderate, severe, or extreme, depending on the patient's BMI: *Mild*: BMI greater than or equal to 17 *Moderate*: BMI between 16 and 16.99 *Severe:* BMI between 15 and 15.99 *Extreme*: BMI less than 15

What mental illness affects males and females equally, worsens with each decade of life, is three times more prevalent in older adults, and is associated with indecisiveness, procrastination, avoidance, and difficulty organizing tasks?

*Correct answer: Hoarding disorder* Hoarding disorder affects males and females equally, worsens with each decade of life, is three times more prevalent in older adults, and is associated with indecisiveness, procrastination, avoidance, and difficulty organizing tasks. Once symptoms begin, hoarding behaviors usually become chronic. About 80 to 90 percent of individuals with hoarding disorder display excessive acquisition, most commonly in the form of excessive buying or excessive collection of free items. A less common type of this disorder is called animal hoarding. In animal hoarding, individuals accumulate a large number of pets and fail to provide them with minimal standards of nutrition, sanitation, or veterinary care.

What virtue is associated with the successful resolution of the psychosocial stage Trust vs. Mistrust?

*Correct answer: Hope* According to Erik Erikson, the virtue of hope is associated with the successful resolution of the psychosocial stage Trust vs. Mistrust. Erikson's stages of psychosocial development are matched with their associated virtue below. *Trust vs. Mistrust*: Hope *Autonomy vs. Shame and Doubt*: Will *Initiative vs. Guilt*: Purpose *Industry vs. Inferiority*: Competence *Identity vs. Role Confusion:* Fidelity *Intimacy vs. Isolation*: Love *Generativity vs. Stagnation:* Care *Ego Integrity vs. Despair*: Wisdom

Rett syndrome is known as the development of specific deficits following a period of normal functioning after birth. You are a nurse practitioner performing an assessment on a new six-year-old female patient. You pay close attention to her mental status and perform a mental status exam. All of the following presentations are expected with Rett syndrome except:

*Correct answer: Inattentiveness* Children with Rett syndrome present with the following: 1. Flat or blunted affect. 2. Impairment of expressive and receptive language 3. Stereotypic hand movements Inattentiveness is a characteristic of children presenting with ADHD, not Rett syndrome.

Which of the following is considered an environmental risk factor?

*Correct answer: Infection* Environmental risk factors include prenatal insults, stress, infections, poor nutrition, exposure to toxins, and physical or sexual abuse. Vaccinations are not considered environmental risk factors, as they are meant to provide supportive prevention of a disease process. Medication management is not considered an environmental risk factor, as they are meant to provide supportive treatment of illnesses and diseases. Exercise regimens are not considered environmental risk factors, because exercise improves health.

Covered under medicaid Part B

*Correct answer: Influenza and pneumonia* Medicare Part B provides coverage for ambulatory practitioner services and physical, occupational, and speech therapy. It also covers medical equipment, diagnostic tests, and some preventative care. Covered preventative services: Pap smears Mammography Screening for colorectal cancer Screening for prostate cancer Screening for cardiovascular disease Screening for diabetes Glaucoma screening Influenza vaccinations

Failure to resolvem psychosocial stages

*Correct answer: Initiative vs. Guilt* Failure to successfully resolve the psychosocial stage Initiative vs. Guilt is associated with the later development of conversion disorder. Erik Erikson's stages of psychosocial development: Trust vs. Mistrust: birth to 12-18 months Autonomy vs. Shame and Doubt: 12-18 months to 3 years Initiative vs. Guilt: 3 to 5-6 years Industry vs. Inferiority: 5-6 years to adolescence Identity vs. Role Confusion: adolescence to adulthood Intimacy vs. Isolation: adulthood Generativity vs. Stagnation: middle adulthood Ego Integrity vs. Despair: late adulthood Failure to resolve Trust vs. Mistrust is associated with psychosis, addictions, and depression. Failure to resolve Autonomy vs. Shame and Doubt is associated with paranoia, obsession, compulsions, and impulsivity. Failure to resolve Industry vs. Inferiority is associated with creative inhibition.

Physical exam findings of Rett syndrome include all of the following except:

*Correct answer: Irregular heart rate* Irregular heart rate is not a key finding in Rett syndrome. Physical exam findings and associated features of Rett syndrome are as follows: Seizures Irregular respirations Scoliosis Loss of purposeful hand skills Stereotypic hand movements *Rule out Autism *

All of the following statements are true about gray matter except

*Correct answer: It is the myelinated axons of neurons* Brain tissue is categorized as either white matter or gray matter. White matter is the myelinated axons of neurons. Gray matter is composed of nerve cell bodies and dendrites. Gray matter is found in the working area of the brain and contains the synapses.

According to Freud, during what psychosexual stage of development are sexual concerns largely unimportant?

*Correct answer: Latency stage* Freud's stages of psychosexual development: Oral: birth to 12-18 months Anal: 12-18 months to 3 years Phallic: 3 to 5-6 years Latency: 5-6 years to adolescence Genital: adolescence to adulthood The latency stage is characterized by a time in which sexual concerns are largely unimportant. During this phase, children establish decisive patterns of adaptive functioning. They develop a sense of industry and a capacity for mastery of objects. This is the phase in which children develop a foundation for mature adult life satisfaction.

As many as 50% of all alcohol-dependent suicide victims have experienced which of the following in the previous year?

*Correct answer: Loss of a close, affectionate relationship* As many as 50% of all alcohol-dependent suicide victims have lost a close, affectionate relationship in the past year. Alcohol-dependent suicide victims tend to be white, middle-aged, unmarried, friendless, socially isolated, and currently drinking. Forty percent have made a previous suicide attempt. Those with comorbid antisocial personality disorder are at an even higher risk of suicide.

Pts w/ OCD also have what other disorder?

*Correct answer: Major depression* Obsessive-compulsive disorder is highly comorbid with other psychiatric conditions: 67% have major depression 25% have social phobia 20-30% have a tic disorder Rates of OCD are also elevated in individuals with schizophrenia, schizoaffective disorder, bipolar disorder, eating disorders, and Tourette syndrome.

An 18-year-old female arrives at your office with a chief complaint of "nervousness." After your full evaluation, you diagnose her with unspecified anxiety disorder and prescribe sertraline. You administer the Hamilton Anxiety Scale to obtain a baseline score, and you plan to monitor her treatment progress by readministering the scale at every follow-up appointment. Her score today is 20. What does this score indicate?

*Correct answer: Moderate anxiety* The Hamilton Anxiety Scale (HAM-A) is the most commonly used clinician-administered anxiety rating scale. It is best used in the evaluation of anxiety severity and tracking the efficacy of anxiety treatments over time. The HAM-A is based on various domains of anxiety including anxious mood, fears, sleep disturbance, somatic complaints, tension, and observed behavior. The severity of each domain is ranked from 0 (not present) to 4 (severe). Scoring is as follows: *Score of 14-17*: Mild anxiety *Score of 18-24*: Moderate anxiety *Score of 25-30*: Severe anxiety

You are a new nurse practitioner opening your own clinic specifically to treat children and adolescents. You have an initial evaluation appointment today with a mother of a six-year-old who is concerned about her daughter's inability to keep up with her schoolwork. You decide to screen for autism spectrum disorder (ASD). All of the following assessments are appropriate for screening this patient for ASD except:

*Correct answer: Modified Checklist for Autism in Toddlers* Although this is an evidence-based and adequate tool, it is for toddlers only; this patient is too old for this assessment tool. Screening for developmental delays is a critical component of assessing for ASD. Checklists for this include the following: Modified Checklist for Autism in Toddlers (Age-Specific) Autism Diagnostic Observation Schedule-Generic (ADOS-G) Ages and Stages Questionnaires Parents may also report the following symptoms: No cooing by age one year, no single words by age 16 months, no two-word phrases by age 24 months Loss of language skills No imaginary play Little interest in playing with other children Extremely short attention span No response when called by name Little or no eye contact Intense tantrums Fixations on single objects Unusually strong resistance to changes in routines Oversensitivity to certain sounds, textures, or smells Appetite or sleep-rest disturbance, or both Self-injurious behavior

A young woman brings her 77-year-old father to a clinic. She believes her father has Alzheimer's disease because he has been depressed and forgetful lately. What are the earliest symptoms of Alzheimer's disease?

*Correct answer: Mood changes* The onset of symptoms in Alzheimer's disease follows this progression: (1) mood changes, (2) cognitive impairment, (3) decline in functional independence, (4) behavioral and motor symptoms. Mood changes are usually manifested as apathy rather than depression and are resistant to antidepressant medications but responsive to cholinesterase inhibitors.

What type of phobia is experienced equally by both genders?

*Correct answer: Needles* There are five categories of phobias: Animal (spiders, insects, dogs, etc.) Natural environment (heights, storms, water, etc.) Blood-injection-injury (needles, invasive medical procedures, blood transfusions, etc.) Situational (elevator, airplane, enclosed places, etc.) Other (situations that may lead to vomiting, loud sounds, costumed characters, etc.) Men and women experience blood-injection-injury phobias at equal rates. Women are more likely than men to experience animal, natural environment, and situational phobias.

A 47-year-old female arrives at the emergency room accompanied by the police, who found her "confused and walking strangely." During your examination, you notice pupil abnormalities, depressed deep tendon reflexes, and a positive Romberg sign. She reports a long history of IV drug abuse. In the past, she was prescribed an antipsychotic, but she states she has not taken that medication for years. What do you suspect?

*Correct answer: Neurosyphilis* Even though syphilis infections have been declining since World War I, it is important to consider them in a differential diagnosis for a variety of psychiatric presentations. Neurosyphilis is syphilis that involves the central nervous system. Symptoms of neurosyphilis include the following: Wide-based gait Positive Romberg sign Loss of vibratory and proprioceptive senses in lower extremities Decreased deep tendon reflexes Pupil abnormalities Tremors Dyscoordination Spasticity in lower extremities The patient has not taken antipsychotic medications for years, so her symptoms are likely unrelated to neuroleptic malignant syndrome (NMS). Neuroleptic malignant syndrome is a rare but life-threatening idiosyncratic reaction to neuroleptic medications that is characterized by fever, muscular rigidity, altered mental status, and autonomic dysfunction. NMS often occurs shortly after the initiation of neuroleptic treatment, or after dose increases. Wernicke's syndrome, also known as Wernicke encephalopathy, is a neurological disease characterized by the clinical triad of confusion, the inability to coordinate voluntary movement (ataxia), and eye (ocular) abnormalities. Her symptoms cannot be explained by Wernicke encephalopathy. Korsakoff syndrome is a chronic memory disorder caused by severe deficiency of thiamine (vitamin B1). Korsakoff syndrome is most commonly caused by alcohol misuse and is often, but not always, preceded by an episode of Wernicke encephalopathy. Her symptoms cannot be explained by Korsakoff psychosis.

A 21-year-old female presents to her primary care provider because she has stopped menstruating. She is a thin, athletic woman with poor dental hygiene and erosion of her tooth enamel. Her weight is slightly below average but within normal limits. She describes herself as "a little weird" about her weight, and she sometimes exercises heavily after eating a big meal. What laboratory findings would you expect with this patient?

*Correct answer: Normal thyroid function* This patient presents with symptoms suggesting a binge eating and purging disorder, such as bulimia. Bulimia does not usually affect thyroid function; therefore, you would expect normal thyroid levels in this patient. Other laboratory findings in patients with bulimia might include the following: Hypokalemia Hypochloremia High serum amylase Hypomagnesemia

You are a new nurse practitioner opening your own clinic specifically to treat children and adolescents. You have an initial evaluation appointment today with a mother of a 15-year-old who is concerned about her daughter's severe weight loss and frequent blackouts. The mother indicates that her daughter is fixated on caloric intake and exercising. During your screening process, the patient endorses that she has not had her menstrual period for eight months. To further understand the progression of a potential eating disorder, you order labs and want to pay specific attention to what?

*Correct answer: Normochromic, normocytic, or general anemia* This patient is presenting with a history of severe weight loss, halted menstrual periods, and blacking out. An important part of the assessment is to check the patient for anemia. Other lab changes to assess for in patients with eating disorders include the following: Leukopenia Neutropenia Thrombocytopenia Hypokalemia Hypomagnesemia Hypoglycemia Decreased LH and FSH Cardiomyopathy (enlarged heart muscle), Wolff Parkinson White (WPW) syndrome, and increased HDL and LDL are not relevant key findings in eating disorders.

What court case determined that the presence of a mental illness alone cannot justify involuntary hospitalization?

*Correct answer: O'Connor vs. Donaldson* The 1976 case O'Connor vs. Donaldson ruled that harmless mentally ill patients cannot be confined against their will if they can survive outside. This case determined that the presence of a mental illness alone cannot justify involuntary hospitalization. In 1979, Rennie vs. Klein determined that patients have the right to refuse any treatment and use an appeal process. In 1981, Roger vs. Oken determined that patients have an absolute right to refuse treatment but that a guardian may authorize their treatment. Durham vs. United States determined that an individual is not criminally responsible if the unlawful act was the product of mental illness. This case is known for originating the insanity defense.

Why are older adults more sensitive to psychotropic medication?

*Correct answer: Older adults have low muscle mass and increased body fat concentration* Psychotropic medications are lipophilic and highly protein-bound. Older adults are more sensitive to psychotropic medications due to their Decreased intracellular water Low muscle mass Decreased protein binding Increased body fat concentration Decreased rate of metabolism

If a patient threatens to commit suicide, what legal doctrine allows a nurse practitioner to involuntarily admit them to a psychiatric hospital?

*Correct answer: Parens patriae* The criteria for involuntary admission comes from the legal doctrines of police power (state as protector) and parens patriae (state as parent). Parens patriae allows the state to intervene and act as a surrogate parent for those unable to care for themselves or who may harm themselves. Police power allows the state to intervene if the patient is a danger to others.

Medicare Coverage Parts

*Correct answer: Part C* The Medicare program has four parts: Part A, Part B, Part C, and Part D. Part A: Coverage for hospitalizations (up to 90 days), skilled nursing facility (up to 100 days), hospice (up to six months for terminally ill), and some home health care Part B: Coverage for ambulatory practitioner service; physical, occupational, and speech therapy; medical equipment; diagnostic tests; and some preventative care Part C: Optional coverage for beneficiaries who can choose to receive all of their health care services through one of the provider organizations covered under the Medicare Advantage plan Part D: Optional coverage for outpatient pharmaceuticals

Which part of Medicare provides optional coverage for outpatient pharmaceuticals?

*Correct answer: Part D* The Medicare program has four parts: Part A, Part B, Part C, and Part D. Part A: Coverage for hospitalizations (up to 90 days), skilled nursing facility (up to 100 days), hospice (up to six months for terminally ill), and some home health care Part B: Coverage for ambulatory practitioner service; physical, occupational, and speech therapy; medical equipment; diagnostic tests; and some preventative care Part C: Optional coverage for beneficiaries who can choose to receive all of their health care services through one of the provider organizations covered under the Medicare Advantage plan Part D: Optional coverage for outpatient pharmaceuticals

Which of the following correctly pairs the pharmacodynamic effect with its definition?

*Correct answer: Partial agonist effect: drug does not fully activate the receptors* Psychiatric medications can cause four different pharmacodynamic effects on cell receptors. Agonist effect: drug binds to receptors and activates a biological response Inverse agonist effect: drug causes the opposite effect of agonist Partial agonist effect: drug does not fully activate the receptors Antagonist effect: drug binds to the receptor but does not activate a biological response

Outcomes management uses variance data to change a system of health care practice. All of the following are sources of variance except:

*Correct answer: Patient survey* Outcomes management is the use of aggregate variance data to change a system of health care practice. Variance is any event that alters patient progress toward expected outcomes. Sources of variance include practitioner behavior (competency), the severity of illness (with high-risk patients), and practice patterns that either expedite care or inhibit delivery of care. Patient surveys are unique to each patient and do not provide variance of practice. The patient is not considered a source of variance when it comes to outcomes management. The focus in outcomes management is system-based and concerned with how the system of care can be improved for better patient outcomes.

In what phase of group therapy will the leader consider the direction and framework of the group?

*Correct answer: Pregroup* In the *pregroup phase*, the leader will consider the direction and framework of the group. In the *forming phase*, goals and expectations are identified and boundaries established. In the *storming phase*, the leader is to allow expression of both positive and negative feelings and assist the group in understanding underlying conflicts and nonproductive behaviors. In the *norming phase*, the leader allows for open and spontaneous communication, and norms are established.

According to Piaget, at what cognitive stage do children develop egocentric thinking?

*Correct answer: Preoperational* Piaget's stages of cognitive development: Infancy (birth to 2 years): Sensorimotor Early Childhood (2 to 5 years): Preoperational Middle Childhood (6 to 11 years): Concrete operational Adolescence (11 to 19 years): Formal operational During the preoperational stage, children develop language, symbolic thinking, and egocentrism. Egocentric thought does not take the viewpoints of others into account.

Which of the following is true regarding the incidence and demographics of those who suffer from or are at risk for bipolar-type disorders?

*Correct answer: Prevalence in males and females is the same* Bipolar disorder is less common than major depressive disorder. Furthermore, 0.7% of the general population is at risk, it affects 2.3 million American adults, it has a mean onset age of early 20s, it may present in childhood or adolescent years, and prevalence is the same for both males and females.

Mean, median, mode, and range are examples of descriptive statistics. Which statistic is defined as the difference between the lowest and highest values?

*Correct answer: Range* Range is the difference between the lowest and highest values. Mode is the value occurring most often. Mean is the average of values. Median is the middle value when the values are listed in numerical order.

Decreased levels of calcium can cause which of the following?

*Correct answer: Renal failure* Decreased levels of calcium can cause acute renal failure. Increased levels of calcium can cause acidosis, Addison's disease, and hyperthyroidism.

What did the Technology Informatics Guiding Education Reform (TIGER) initiative successfully define for nurse practitioners?

*Correct answer: Required technology curriculum for colleges* The Technology Informatics Guiding Education Reform (TIGER) initiative developed a 10-year plan for nursing's path toward computer and information literacy. It involved more than 1,100 nursing content experts, and it took three years to complete. This initiative successfully defined the basic technology competencies and required curriculum for nurse practitioner education. The TIGER initiative declared that it is a nurse practitioner's responsibility to understand and shape the landscape of health care technology in order to improve access, quality, and the patient experience.

You are treating a 45-year-old patient who has been diagnosed with schizophrenia and treated with thioridazine (Mellaril) for the past 15 years. You decide to switch the patient to a different antipsychotic due to what serious long-term consequence of high-dose thioridazine?

*Correct answer: Retinal pigmentation* Retinal pigmentation can occur in patients prescribed >1,000 mg of thioridazine. Even when thioridazine is stopped, this side effect may persist and lead to blindness. Hyperprolactinemia, priapism, and tardive dyskinesia can occur as side effects of any antipsychotic medication

Substance use screening tool interpretation is an important part of advanced assessment and of the role of the psychiatric-mental health nurse practitioner. The most common tool used to assess for alcoholism in geriatric populations is:

*Correct answer: S-MAST* The S-MAST, also known as the Michigan Alcoholism Screening Test for Geriatrics, is a clinical assessment tool used to assess and screen for alcoholism in geriatric populations. The COWS (Clinical Opiate Withdrawal Scale) assessment is used to assess the withdrawal severity of opiates. The CIWA assessment is used to assess the withdrawal severity of alcohol and/or benzodiazepines. The AUDIT assessment is also an alcohol use assessment tool, less commonly used than CIWA.

A nurse practitioner volunteering on a disaster response team following a terrorist bombing is engaging in what type of public health prevention strategy?

*Correct answer: Secondary prevention* A nurse practitioner volunteering on a disaster response team following a terrorist bombing is engaging in a secondary prevention strategy. Secondary prevention is aimed at decreasing the prevalence of mental disorder. In this scenario, the psychiatric nurse practitioner is providing prompt and early screening for acute stress reactions following the bombing.

A 31-year-old male diagnosed with depression undergoes an fMRI. His test reveals increased activity in the ventromedial prefrontal cortex. Which class of medication aids in pyramidal cell functioning in the prefrontal cortex by preventing the excessive release of glutamate?

*Correct answer: Serotonin antagonist/reuptake inhibitors* Serotonin antagonist/reuptake inhibitors block 5HT2A receptors. This action inhibits glutamate release in the prefrontal cortex of the brain. Neither SSRIs, SNRIs, nor NDRIs block 5HT2A receptors. two main SARIs prescribed in the United States are:3 Trazodone (previous trade names include Desyrel, Oleptro). Nefazodone (previously marketed as Serzone).

Commonly seen in Japan and Korea, the syndrome of taijin kyofusho is a form of what mental illness?

*Correct answer: Social phobia* Commonly seen in Japan and Korea, the syndrome of taijin kyofusho is a form of social phobia. It is characterized by a fear of social evaluation. The patient often reports that they believe they are making other people uncomfortable. For example, a patient might state, "My gaze upsets people, so they avoid me." This fear also has some traits of body dysmorphic disorder (in that the patient may believe they possess an offensive body odor) and it can accelerate to a delusional intensity.

What focused method of therapy is being used when a provider seeks to view symptoms as metaphors in the hierarchal structure of the family unit?

*Correct answer: Strategic therapy* Strategic therapy, originated by Jay Haley, is a method in which interventions are both symptom- and problem-focused. Symptoms are viewed as metaphors and reflect problems in the hierarchal structure. Symptoms are a way to communicate metaphorically within the family. Solution-focused therapy, originated by Steve deShazer, Bill O'Hanlon, and Insoo Berg, is a method where the provider focuses on effective resolution of problems through cognitive problem-solving and use of personal resources as strengths. Structural family therapy, originated by Salvador Minuchin, is a model that places emphasis on how, when, and to whom family members relate in order to understand and then change the family's structure. Family systems therapy, originated by Murray Bowen, is used to increase the family's awareness of each member's function within the family and to increase levels of self-differentiation.

Levels of prevention

*Correct answer: Tertiary prevention* . *Primary prevention* is aimed at decreasing the number of new cases of mental disorders. An example of primary prevention is stress management classes for graduate students. Secondary prevention is aimed at decreasing the number of existing cases of mental disorder. Examples of secondary prevention include telephone hotlines, crisis intervention, and disaster response. *Tertiary prevention*: is aimed at decreasing the disability and severity of a mental health disorder. Providing social skills education for a group of intellectually disabled teenagers is an example of tertiary prevention

There are four levels of systems: microsystems, macrosystems, megasystems, and metasystems. In health care, what is an example of a megasystem?

*Correct answer: The American health care system* Organizational knowledge and systems thinking add an imperative dimension to nursing science. Nurse practitioners understand that organizations are systems in motion with complex relationships between many different parts. There are four levels of systems: microsystems, macrosystems, megasystems, and metasystems. Microsystem: direct patient care Macrosystem: hospitals, skilled nursing facilities, clinics Megasystem: American health care system Metasystem: economic, political, and social level of society

Which of the following patient characteristics would suggest that their psychiatric symptoms are actually due to a physical illness?

*Correct answer: The patient is more than 40 years old* Signs, symptoms, and historical information can suggest an underlying physical condition. A physical cause for psychiatric symptoms is most likely if the patient is having their first episode of mental illness or is more than 40 years old. Persons younger than 20 can develop psychiatric conditions, and estimates suggest that 1 in 5 people will experience a mental illness. A decreased appetite or early morning waking could indicate major depression.

Regarding nurse practitioner standards of practice, all of the following statements are true except:

*Correct answer: There are broad variations from state to state due to outdated legislation* Nurse practitioner scope of practice: Defines the nurse practitioner's roles and actions Identifies competencies assumed to be held by all nurse practitioners who function in a particular role Has broad variations from state to state due to outdated legislation Nurse practitioner standards of practice: Give authoritative statements regarding the quality and type of practice that should be provided Offer a way to judge the nature of care provided Reflect the expectation for the care that should be provided to patients Reflect professional agreement focused on the minimum levels of acceptable performance Can be used to legally describe the standard of care May be either precise or general guidelines Reflect one guideline applicable to all psychiatric nurse practitioners regardless of their practice location

A 73-year-old Caucasian male with a diagnosis of Alzheimer's disease presents to your clinic. He currently smokes two packs of cigarettes per day and declines help with smoking cessation. Which medication for Alzheimer's disease is contraindicated due to his smoking habit?

*Correct answer: There are no contraindications due to smoking for these medications* You can choose any cholinesterase inhibitor to prescribe to this patient. There are no contraindications due to smoking for these medications. *Galantamine* might be the best choice for this patient because of its dual actions of (1) inhibiting acetylcholinesterase and (2) positive allosteric modulation of nicotinic cholinergic receptors.

Up to 30 percent of individuals with obsessive-compulsive disorder also develop what mental illness?

*Correct answer: Tic disorder* Up to 30 percent of individuals with obsessive-compulsive disorder (OCD) will develop a tic disorder. This is most commonly seen in males who develop OCD as a child.

Which of the following is most likely to be a goal of family systems therapy?

*Correct answer: To increase levels of self-differentiation* Family systems therapy was developed by Murray Bowen, who believed that an individual's problematic behavior may serve a function or purpose for the family. This therapy focuses on the chronic anxiety within families. Treatment goals of family systems therapy include increasing self-differentiation, which helps family members learn that their self-worth is not dependent on external relationships, circumstances, or occurrences. Changing family structure is a goal of structural family therapy. Developing nurturing communication is a goal of experiential therapy. Helping family members behave in ways that will not perpetuate the problem behavior is a goal of strategic therapy.

What is systems thinking?

*Correct answer: To produce what it produces* In systems thinking, the belief is that a system is perfectly designed to produce what it produces. The belief is that whatever comes from a system is what the system was designed to produce, regardless of whether the design of the system was planned or unplanned and whether the results were intended or unintended.

As a nurse practitioner, you are responsible for assessing cranial nerves as a part of the neurological exam. What cranial nerve assessment tests for tactile perception of the facial skin?

*Correct answer: Trigeminal nerve (CN V)* *Olfactory nerve (CN I)* The olfactory nerve transmits information to the brain regarding a person's sense of smell. This cranial nerve assessment is performed by having the client close their eyes to identify familiar odors. *Optic nerve (CN II)* The optic nerve transmits information to the brain regarding a person's vision. *Oculomotor nerve (CN III)* The oculomotor nerve helps control muscle movements of the eyes. The oculomotor nerve provides movement to most of the muscles that move the eyeball and upper eyelid, known as extraocular muscles. The oculomotor nerve also helps with involuntary functions of the eye. *Trochlear nerve (CN IV)* The trochlear nerve is also involved in eye movement. The trochlear nerve, like the oculomotor nerve, originates in the midbrain. It powers the contralateral superior oblique muscle that allows the eye to point downward and inward. *Trigeminal nerve (CN V)* The trigeminal nerve is the largest cranial nerve and has both motor and sensory functions. Its motor functions help a person to chew and clench the teeth and gives sensation to muscles in the tympanic membrane of the ear. This test is performed by touching the face and corneal reflex of the eye with a wisp of cotton and pin-pricking the skin and mucosa to test touch. *Abducens nerve (CN VI)* The abducens nerve also helps control eye movements. It helps the lateral rectus muscle, which is one of the extraocular muscles, to turn the gaze outward. *Facial nerve (CN VII)* The facial nerve functions to produce facial expressions and has both motor and sensory functions. *Vestibulocochlear nerve (CN VIII)* The vestibulocochlear nerve is involved with a person's hearing and balance. It is assessed by checking the client's hearing using an audiometer or by simply whispering in the client's ear. Hearing loss is tested with the Weber and Rinne tests. *Glossopharyngeal nerve (CN IX)* The glossopharyngeal nerve possesses both motor and sensory functions. It is involved with the sensation of taste for the back of the tongue. *Vagus nerve (CN X)* The vagus nerve has a range of functions, providing motor, sensory, and parasympathetic functions. It plays a role in taste sensation, provides movement to the throat and soft palate, and regulates heart rhythm. *Accessory nerve (CN XI)* HeThe accessory nerve provides motor function to the neck. It controls muscles that allow a person to rotate, extend, and flex the neck and shoulders. The cranial part of the accessory nerve combines with the vagus nerve. *Hypoglossal nerve (CN XII)* The hypoglossal nerve is a motor nerve that supplies the tongue muscles.

Clinically significant side effects of monoamine oxidase inhibitors include insomnia, weight gain, sexual dysfunction, and dizziness.

*Correct answer: True* Clinically significant side effects of monoamine oxidase inhibitors include insomnia, weight gain, sexual dysfunction, dizziness, hypertensive crisis, lightheadedness, and anticholinergic side effects.

Mental status exam findings of those suffering from major depressive disorders include having an unkept appearance; being tired-looking; wearing dark-colored, loose-fitting clothes; and having significant weight changes.

*Correct answer: True* The following are specific attributes often found in those suffering from major depressive disorders: Unkept appearance Tired appearance Wearing dark-colored clothing Wearing loose-fitting clothing Significant weight gain or loss

Strategies for reducing homelessness include all of the following except:

*Correct answer: Urgent and acute treatment of symptoms of homeless individuals who are mentally ill* Although urgent and acute treatment of symptoms of homeless individuals is a part of treatment, it has little to no lasting effect when this is the only treatment provided, as it is not sustainable and not supportive in the long-term goal of reducing homelessness. Strategies for reducing homelessness include the following: Outreach: Introducing services to homeless persons with serious mental illness in various settings; building an empathetic, consistent, and caring relationship to provide treatment. Integrated care: Treatment combining mental health and medical care to improve overall functioning in the community. Can do this by providing colocation of mental health and primary care at a single site. Supportive services in housing: Moving homeless persons with serious mental illness directly to independent housing with support and intensive attention. Prevention: Beginning with discharge planning in inpatient settings, provide resources for mental health care, housing, transitioning service, and follow-up.

For an act to be considered criminal, it must have two components: actus reus and mens rea. What is actus reus?

*Correct answer: Voluntary conduct* For an act to be considered criminal, it must have two components: actus reus and mens rea. Actus reus refers to voluntary conduct. Voluntary conduct is deemed impossible if the offender's mental status is deficient, abnormal, or diseased in a way that inhibits rational intent. Mens rea refers to evil intent. Evil intent is defined as the resolve to do harm. Neither behavior nor intent alone is enough to convict a person of a crime.

Systemic effects of hypernatremia include all of the following except:

*Correct answer: Weight gain* Other presentations of hypernatremia: Convulsions Pulmonary edema Dry mucous membranes Tachycardia Low jugular venous pressure Restlessness Presentations of hyponatremia: Lethargy Headache Confusion Apprehension Seizures Coma Hypotension Tachycardia Decreased urine output Weight gain Edema Ascites Jugular vein distension

A patient presents with oculomotor disturbances, cerebellar ataxia, and mental confusion. What psychiatric emergency would you suspect?

*Correct answer: Wernicke's encephalopathy* Oculomotor disturbances, cerebellar ataxia, and mental confusion suggest a diagnosis of Wernicke's encephalopathy. The nurse practitioner should give thiamine 100 mg IV or IM and conduct a further evaluation. Other presentations A patient experiencing bereavement might present with feelings of guilt, irritability, insomnia, and somatic complaints. Cocaine intoxication would present with paranoia, violence, severe anxiety, tachycardia, hypertension, and possibly myocardial infarction. Classic signs of alcohol intoxication include disinhibited behavior, slurred speech, discoordination, and sedation at high doses.

Descriptive statistics

*Correct answer: p-value* Descriptive statistics are used to describe the basic features of the data in a study. They are numerical values that summarize, organize, and describe observations. Examples include mean, standard deviation, and variance. Inferential statistics are numerical values that enable a researcher to draw conclusions that extend beyond the immediate data alone. Examples include t-test, analysis of variance, Pearson's r correlation, probability, and p-value.

Which inferential statistic determines whether the means of two groups are statistically different from each other?

*Correct answer: t-test* Inferential statistics are numerical values that enable one to reach conclusions that extend beyond the immediate data alone. Examples include: *t-test*: assesses whether the means of two groups are statistically different from each other *Analysis of variance (ANOVA)*: tests the difference among three or more groups *Pearson's r correlation*: tests the relationship between two variables *Probability*: the likelihood of an event occurring *p-value (level of significance)*: describes the probability of a particular result occurring by chance alone. For example, if p = 0.01, there is a 1% probability of obtaining the result by chance alone.

Which of the following findings on MRI and PET scans would be uncommon in individuals with schizophrenia?

*Decrease size of cerebral ventricles* Individuals w/ schizophrenia have enlarged cerebral ventricles. There is decreased electrical activity in frontal lobes, decreased glutumate and GABA release, and decreased total brain volume.

In distinguishing between dementia and pseudodementia which of the following findings would be expected in a pt with pseudodementia?

*Don't know" answers typical* Dont know answers are typical in pseudodementia constrated to frequent, "near miss" answers in dementia. Attention and concentration are well preserved in pseudodementia contrasted to faulty attention and concentration in dementia Nocturnal accentuation of dysfunction is common in dementia, but it is uncommon in pseudodementia Patients w/ dementia struggle to perform tasks contrasts by pseduodementia where patients make little effort to perform even simple tasks.

Which of the following is not the responsibility of IRB?

*Ensure that animal studies are conducted using humane methods* IRBs were established to protect the rights and welfare of human research participants. Animal studies are not within the purview.

Mrs. Kemp is voluntarily admitted to the hospital. After 24 hours, she states she wishes to leave because "this place can't help me." The best nursing action that reflects the legal right of this client is

*Explain that the client cannot leave until you complete further assessment.* Almost every state allows for a brief period of detainment to assess a client for dangerousness to self or other before allowing the client to leave hospital settings, even if admission is voluntary

According to Freud, successful resolution of which psychosexual stage provides the basis for the development of personal autonomy and capacity for self-confidence?

*Freud's stages of psychosexual development:* *Oral*: birth to 12-18 months *Anal*: 12-18 months to 3 years *Phallic*: 3 to 5-6 years *Latency*: 5-6 years to adolescence *Genital*: adolescence to adulthood Successful resolution of the anal stage of psychosexual development provides the basis for the development of personal autonomy and capacity for self-confidence. Failure to resolve the anal stage results in pathological traits of excessive orderliness, stubbornness, willfulness, frugality, and parsimony. Anal characteristics and defenses are typically seen in obsessive-compulsive disorders.

Making an evidence based decision regarding implementation of an intervention includes current research evidence all of the following except ?

*Generalizability of findings* Generalizability refers to a particular study's findings and to what extent those findings are applicable to patient's in other practice settings.

You notice that you hve begun to care less about your pts needs. You are tired, unhappy, and unsatisfied in your job. You talk to your office confidant, another psych NP about feeling possible compassion fatigue. He indicates that while he can understand your concerns based on your assessment, but he does not believe have compassion fatigue because he has not observed what?

*Heavily criticizing others during meetings, sitting in your office alone all the time, dressed in the same outfit, denial about your problems* Symptoms include denial about problems, excessive blaming, holding in emotions, isolating from others, unusual amount of complaints, complaining about administrative functions, substance abuse, compulsive spending, overeating, gambling or sexual addiction and poor self-care

A WBC of 4,ooo in a patient taking clozapine would prompt the PMHNP to take which of the following actions.

*Institute a twice-weekly complete blood count w. differentials and monitor closely* D/c clozapine when WBC of 2000-3000 or granulocytes of 1,000 to 1,500 for agranulocytosis and severely compromised immune system, At a WBC of 4,00 recommendation is to closely monitor CBC w/diff twice a week while pt may continue clozapine in the absence of any other signs or symptoms.

When evaluating a 5-year-old child with language deficits, which of the following is a key indicator of differentiating autistic DO from a mixed receptive-expressive language disorder?

*Language abnormalities such as echolalia are common in Autism* Language abnormalities such as echolalia and stereotyped phrases out of context are more common in autistic DO and are infrequent in mixed receptive expressive language disorders. In both disorders 25% of cases have fam hx of speech delays or language problems. Intelligence levels for both disorders can range from mild to severe, although in autistic disorders more severe impairment occurs more often. Imaginative play is usually present in the mixed receptive-expressive language disorder and either absent or rudimentary in autism. It is not the predominant for of expression for either disorder.

When suspecting a patient with NMS, which lab finding values would help confirm dx?

*Leukocytosis and elevated creatine phosphokinase* With NMS a patient has an elevated CPK d/t skeletal muscle breakdown and elevated white blood cell count*

A 43-year old male treated for BP 1 on lithium. Which of the following hematologic changes in associated with lithium?

*Leukocytosis* While the MOA of lithium is not clear, the use of LI can increase white blood cell counts and therefore CBC should be monitored in patients with lithium.

Example of a screening tool that measures severity and tracks changes in specific symptoms?

*MMSE* The MMSE is an example of screening tool that measures that severity and tracking change in specific symptoms. It is used to provide an assessment of broad array of cognitive functioning, including orientation, attention, memory, construction, and language. It is commonly used to screen for dementia and in following the progression of dementia over time.

Appraisal of the pts suicidal ideation, plan, method, intent, and access to implement plan would be documented in which part of the psych eval.

*MSE* Current si, plan, method, intent, and access to implement plan is documented in the MSE under the section for thought content.

When working w/ adolescent who have poor affect regulation and limited response flexibility, therapists strive to provide a therapeutic frame open enough for changes and consistent enough for stability and safety. Which of the following responses characterizes such a therapeutic frame

*Mindful, reflective, intentional* *Genuine* *In every one, not just adolescents* Core tenants of family, individual, and group therapy.

After 2 weeks of treating a client with psychosis, the client develops symptoms of NMS. The following symptoms are different from serotonin syndrome in?

*Mutism, leukocytosis, myoglobinuria* NMS can be differentiated from serotonin syndrome by the presence of *mutism, leukocytosis, dysphagia, and lab findings of muscle injury, elevated CPK, myoglobinuria, or rhabdo*

The PMHNP understands which factor to be the most important in therapeutic communication

*Non-verbal communication* Gestures, facial expressions, and body language actually communicate more than verbal messages.

When a clinical trial fails to reject the null hypothesis (when pval is > 0.5) the investigator may have missed a true intervention effect. One should consider the following to avoid a type II error.

*Power* Type II error occurs when investigators falsely rejects the null and concludes that NO RELATIONSHIP exists, between variables, in fact, a relationship DOES exist. This occurs when there is insufficient power to detect the different between variables and is linked to the adequacy of the sample size. if a sample size is too small, the study will have have enough power to detect differences between groups.

Which area of the brain has been implicated to modulate pain and can help explain how transcranial magnetic stimulation decrease pain?

*Prefrontal cortex* Modulates pain and the perception of pain; loss of gray matter has been associated with chronic pain, possibly d/t predisposition, or chornic medication use may be toxic to gray matter

A community has an unusually high incidence of depression and drug use among the teen- age population. The public health nurses decide to address this problem, in part, by modifying the environment and strengthening the capacities of families to prevent the development of new cases of depression and drug use. What is this is an example of?

*Primary prevention* This action focuses on the interventions designed to reduce the incidence of new cases of disease.

Mr. Smithers, an involuntarily hospitalized patient experiencing psychotic symptoms, refuses to take any of his ordered medication because he believes "Jesus Christ told me I am the prophet and must fast for a year." Your actions should be based on your knowledge of which of the following?

*Psychiatric Clients can refuse treatment.* As with any client, psychiatric clients can refuse treatment unless a legal process resulting in involuntary commitment or mandatory court order for treatment has been obtaind.

Mr. Thompson has been forgetful lately, for example, forgetting where he has placed his keys or what time appointments are scheduled, and he has stated that he thinks these are just random behaviors that have no particular meaning. Which Freudian-based psychodynamic principle assumes that all behavior and actions are purposeful?

*Psychic determinism of people* The psychic determinism principle states that all behavior has purpose and meaning, often unconscious in nature, and that no behaviors occur randomly.

A ten-yea-old child lost her father to an unexpected hr attack. normally confident, social, and excellent student. Her teachers and mother report that she has withdrwan, become disinterested in schoomates and school work, and easily becomes tearful if anyone mentions her father. What non-pharmacologic interventions would be most helpful at this time to facilitate grieving and loss.

*Psychoeducation group* Normalizing the experience of grief and loss with peers with family similar losses in psychoed group format builds on the value of peer support, installation of hope, and universality that she is not alone in this experience.

The core values that underlie advanced practice nursing and culturally competent care is

*Respect, advocacy, partnership*

An example of a mature, healthy defense mechanism is?

*Sublimation* Suppression is the only defense mechanism listed in which the client channels conflicting energies into growth-promoting activities.

Non-pharmacological methods for anxiety, depressed mood, low energy, little motivation, diminished sex drive and work-related stress for a patient who does not want to be on hormone replacement therapy or anti-depressants?

*Tai chi, dietary soy, flaxseed*

When working w. avoudant patient with a hx of truama, what type of communication techniques are helpful?

*Techniques to increase arousal* Activation is needed to allow memories stored in the amygdala to be processed

Per HIPAA an NP is who is an independent, single practitioner is considered a "covered entity" because ?

*The NP Transmits protected health info in electronic forms* Having the ability to transmit protected health info qualifies the independent NP as a covered entity and requires the complete package of HIPAA required protection in place to protect patient info.

Which of the following would not be advised to divorced parents to facilitate recovery for their children?

*The divorced couple must provide an age-appropriate truthful explanation of why they are getting divorced?* Divorce recovery for children does not necessarily require an explanation of why the parents divorced, particularly if circumstances are damaging or affix blame to one parent over the other (infidelity, sexual orientation, drug use). Despite framing in age-appropriate explanations. Avoiding arguments, demonstrating fair and consistent behavior toward the child, and continuing respectful communication despite a child's anger are recognized as principles of divorce recovery.

As a psych NP you are asked to "case manage" your patients for more than psych issues. Your 68 yr old pt with medicare as the primary insurance states that varicose veins are an issue and that treatment is needed. You know your sister is a vascular vein surgeon in your city and refer the patient to her. You have violated?

*The stark Law* Per the stark law, you are not permitted to refer to fam members for treatment of your patients as this could be seen as fraudulent by federal gov/medicare. Start law guidelines Chapter IV-centers for medicare and medicaid services of department of health and human services part 411- limits medicare payments to siblings.

Which of the following would not be recommended psychotherapy practice w/ a pt diagnosed with borderline?

*Therapist as a passive listener* In psychotherapy for pts w/ BPD,it is important that the therapist be an active and directive, not passive.

8 yr old male, presents w his foster mom for an initial eval. When interviewing the mom alone, it is confirmed that johnny was taken out of his home dt sexual abuse by stepfather. As the treating NP, you know which of the following are evidenced based therapy for Johny?

*Trauma focused CBT* Trauma focused CBT is the minimum of therapy standard with a child who has a hx of trauma*

When child, adult partner, or elder abuse is suspected the PMHNP needs to conduct an abuse assessment screen in privacy, away from the partner, the child's parent or guardians, or the elder person's relative or companion, simple, direction questions in a non-judgmental interview are indicated. Which of the following would be an appropriate abuse screening questions.

*Within the past year, have you been hit, slapped, kicked or otherwise, physically hurt by someone?* This response is one of the four items on a standardized abuse assessment screen

When counseling a woman w/ children who is living in a violent domestic relationship, the PMNP needs to advise that?

*Written material about shelters found by the perpetrator may trigger violence* Although anxiety, depression, dissociation and PTSD are common in survivors of domestic violence, the greater priority is to develop safety plan. Restraining orders cannot always protect someone in domestic violence situations and in some cases may exacerbate the situation. Shelters do not always provided care for children, especially if they are over the age of 14. Do not provide information that may not be true. It is true that written material about shelters and dv, if found by perpetrator can provide an excuse for further battering or violence, The woman needs to be advised to either not take printer material or insure that it cannot be found bt the perpetrator. it is safer to provider shelter numbers that attend her specific needs (housing with children) without additional printer info.

A 31 yr old married woman w/ hx of anorexia, binging-purging type, during her teens and early 20s, is pregnant. She has had two prior spontaneous abortions in late 20's/ She is worried whether this pregnancy may trigger recurrence of her prior eating DO, adverse impact fetal development, or affect the likelihood of full-term delivery. At 5'7, she maintains a weight of 120 lbs by running 6-10 miles daily, condition for marathons and 2,00 calorie daily vegan diets. What modifications would reduce risk of excascerbating eating DO and promote normal fetal development and full-term delivery?

*increase daily caloric intake by 500 calories, reduce running to 2-4 miles daily, alternate with stretching and yoga in second and third trimesters, add prenatal vitamin and bitamin b-12 supplement, plan 28-40# gradual weight over the course of pregnancy. 120lbs for 5'7 her BMI is 18.8 normal range is 19.8-26.0 Low pregnancy BMI and failure to gain recommended weight during pregnancy are associated with risk for spontaneous abortions, pre-term delivery, and low birth weight in infants. Recommended caloric intake for women pregnant women during pregnancy varies from additional 150 calories in first trimester, 300 calories in second trimester, and 500 calories in third before account for strenous daily exercise. Recommended weight gain for pregnant women of normal BMI is #25-35 and 28-40 for women with low BMI. THe vegan diet lacks animal fat, the only dietary source of vitamin B-12. Additional of pre-natal supplments of b-12 is indicate for vegan

When treating as 12-year-old boy for ADHD, the NP notices which of the following physical features that raise concerns for genetic evaluation for Fragile X Syndrome?

*long head, and ears, short stature, hyperextensible joints* Etiological factors in mental retardation can be primarily genetic, developmental, acquire or a combo. Genetic causes included chromosomal and inherited conditions. Many of these genetic disorders have characteristic physical features that warrant genetic testing to comfirm dx. Fragile X syndrome occurs in about 1 of every 1,000 males and 1 of every 2,000 females. The typical phenotype includes a large, long head and ears, short stature, hyperextensible joints, and postpubertal macroorchidism . The mental retardation ranges from mild to severe. There is high rate of comorbid ADHD, learning disorders, such as autism. Cafe Au Lait spots and neurofibromas are common in Von Reclinhausen's disease. Obesity small stature, small hands and feet, hypotonia and hypogonadism are characteristics of prader willi syndrome. Small head, short palpebral fissure, inner epicanthal folds are characteristic of fetal alcohol syndrome

In counseling a 23 yr old, hispanic mother who brought her 4-year-old son into the clinic for mal-de-ojo, with symptoms of fitful sleep, diarrhea, vomiting, and fever, the PMHNP does what?

*respect the mother's understanding of the illness* The NP is culturally competent and respects culture-bound beliefs before proceeding with further steps in the assessment and appropriate interventions. The NP would negotiate to achieve a culturally competent outcome for care.

You see a pt for a routine med visit. At the end of the session, the pt asks questions and sessions ends up being 50 mins in length. You normally charge for the 30 mins appointment but instead you charge for one hour. The one hour appointment includes a full body assessment that you did not perform. This violation is known as?

*upcoding* Upcoding is a fraudulent practice in which the provider services are billed at higher procedure codes than were actually performed, resulting a higher payment by medicare or other third parties.

In evaluating the functional of 68-year-old woman who has depression, the instrumental activities of daily living scale includes which of the following activities?

*using telephone* Instructmental activities of daily living IADls include those that facilitate or enhance the performance of ADLs (shopping, using the telephone, and using transportation). Functional activities or activities of daily living (ADLS) are the activities necessary for the sel-fcare (bathing toileting dressing and transferring). Cognitive exec functioning includes higher-order activities such as balancing a check book, assembling rax records, filling taxes, and driving.

Violence directed toward women by an intimate partner is a serious health problem worldwide. Which of the following women has the greatest risk factors for being the victim of violence?

17-years-old, single native alaska woman living with employed boyfriend in Galena Alaska. Violence directed at women cuts across all ethnic, racial, and socioeconomic lines. However, studies show that Asian/pacific islander women report the lowest rates of IPV and african american and native american/alaska native women report the highest rates of DV Evidence suggests that single, divorced, and separated women are at greater risk than married women. Other women at higher risk of abuse include pregnant women, lower income women, less educated women, and women in relationships with income

One of the health care changes that has occurred as a result of the affordable care act (ACA) is that doctors/hospitals/clinic groups or health systems are coming together and assum- ing the responsibility for quality care to large groups of individuals insured by Medicare. The review Questions 389 health care clinics/systems doctors or hospitals that join together are called which of the following?

Accountable Care Organizations (ACOs) Are a groups of doctors or other providers who voluntarily come togrther and assume the care provided to medicare patients.

The MMSE is a brief, standardized screening tool designed for

Adults to quantify cognitive status THE MMSE is a brief measure of cognitive status in adults that assessess orientation, attention, calculation, recall, language, and motor skills. It can be used to screen for cognitive impiarment to estimate the severity of cognitive impairment at given points in time, to follow the course of cognitive changes in an individual over time, and to document an individual's response to treatment.

As a PMHNP, you understand the genetic factors that contribute to psychiatric and personal- ity disorders. Persons who develop antisocial personality disorder often are raised in families with high rates of which of the following?

Alcohol use disorders Being raised in alcoholic family increases the likelihood of chaos, unprediactability, and lack of rules and order, leading to higher rates of developing antisocial personality disorder.

The neurobiological theory of schizophrenia implicates three areas of neurobiological functioning: genetics, neurodevelopment, and neurobiological deficits. False True Hide ExplanationFlag Question

All three areas of development are implicated in schizophrenia. There are genetic and biological components that contribute to schizophrenia as well as development of the brain and environmental factors that contribute to neurodevelopment.

Treatment for Aids dementia complex

Anti-retroviral therapy

Anticholinergic, antiadrenergic, antihistaminergic side effects

Anticholinergic side effects include dry mouth, sedation, blurred vision, fever, urinary retention, agitation, confusion, and seizures. Orthostatic hypotension is an antiadrenergic side effect. Weight gain is an antihistaminergic side effect.

The five A's for Health Behavior Change include the following:

Assess, Advise, agree, assist, arrange

Attachment relations between infant and mother are considered critical by neurobehavioral researchers because

Attachments to significant relationships (mother, father, caregiver) that the young child begins to find ways for regulation and expression emotions. Self-regulation of emotions is an emotional skill that continues to develop.

Which mental illness is more common in high-income countries than in low-income countries?

Bipolar Disorder

Which of the followings meds has a unique MAO in that it is both a dopamine and NE reuptake inhibitor?

Buproprion boosts neurotransmitters/NE and noradrenaline and dopamine

Systemic effects of hyponatremia include which of the following?

Confusion

Delirium is known to be associated with high mortality and morbidity. Delirium is presenting more and more in the emergency room setting and is often mistaken for a psychotic emergency or event. As a nurse practitioner, you will be responsible for assessing individuals for psychosis versus delirium. Which of the following mnemonics is the correct one for assessing for delirium?

Correct answer: D = Drugs E = Electrolyte abnormality L = Low oxygen saturation I = Infection R = Reduced sensory input I = Intracranial U = Urinary or renal retention M = Myocardial The other options are not correct mnemonics for assessing delirium.

Calcium values can be increased during treatment with anticonvulsants, aspirin, corticosteroids, heparin, and oral contraceptives (TRUE OR FALSE)

Correct answer: False Calcium values can be decreased during treatment with anticonvulsants, aspirin, corticosteroids, heparin and oral contraceptives.

Failure to Resolve Erikson's Stages of identity vs role confusion is linked to?

Correct answer: Identity vs. Role Confusion Failure to successfully resolve the psychosocial stage Identity vs. Role Confusion is linked to delinquent behavior, borderline psychotic episodes, and gender-related identity disorders. Erik Erikson's stages of psychosocial development: Trust vs. Mistrust: birth to 12-18 months *Autonomy vs. Shame and Doubt*: 12-18 months to 3 years *Initiative vs. Guilt*: 3 to 5-6 years *Industry vs. Inferiority*: 5-6 years to adolescence *Identity vs. Role Confusion*: adolescence to adulthood *Intimacy vs. Isolation*: adulthood *Generativity vs. Stagnation*: middle adulthood *Ego Integrity vs. Despair*: late adulthood Failure to resolve the *Industry vs. Inferiority* stage can contribute to creative inhibition. Failure to resolve the *Initiative vs. Guilt* stage can lead to the development of conversion disorder, phobias, and psychosomatic disorders. Failure to resolve the *Autonomy vs. Shame and Doubt* phase can lead to the development of paranoia, obsessions, compulsions.

When treating older adults, you should keep in mind that they are more sensitive to issues of drug toxicity because of which of the following reasons?

Decreased body fat Older adults usually decreased protein levels. Most medications are highly protein-bound. It is the free concentration of the drug that is active; the bound concentration of the drug is inert. Thus, with decreased protein available for binding, more free active drug remains in the body, which then predisposes older adults to toxicity.

Decreased levels of sodium (Na) can cause which of the following?

Decreased, levels of sodium (Na) can cause Addison's disease, renal disorders, and gastrointestinal fluid loss from vomiting and diarrhea.

Persons with obsessive-compulsive personality disorder often use isolation as a defense mechanism. Which of the following examples best describes isolation as a defense mechanism?

Describing information with very little affect variation. Isolation is a defense mechanism often used by people w/ OCPD and has to do with affect and emotion rather than getting out and being social.

Which is true about pharmacologic treatment of anxiety in older adults?

Drugs that are highly oxidized are more unpredictable than drugs that are mostly conjugated. Liver enzymes functioning diminishes as we age.

What four elements need to be present for a malpractice lawsuit to be filed?

Duty of care, breach of standard of care, injury must be r/t to breach of the standard of care.

In order to assess a patient's potential to harm others in addition to themselves, a nurse practitioner might ask what question?

Explanation Details *Correct answer: Are there others who you think may be responsible for what you are experiencing?* To assess a patient's potential to harm others in addition to themselves, nurse practitioners should ask the following questions: Are there others who you think may be responsible for what you are experiencing? Are you having thoughts of harming others? Who? Are there other people you want to die with you? Are there others who you think would be unable to go on without you? For patients who present with thoughts about wanting to harm themselves, nurse practitioners could consider asking: How close have you come to acting on those thoughts? How likely do you think it is that you will act on them in the future? What do you envision happening if you actually killed yourself? Have you made a specific plan to harm yourself? Are guns or other weapons available to you? Have you made particular preparations for your death?

Which of the following would raise concern in an annual sport exam for a 15-year-old girl?

Height 67 inches, weight 102 lbs. BMI is 16 which is less than 5th perctile for her, placing her in underweight/ Formula: weight (kg) / [height (m)]2 Calculation: [weight (kg) / height (cm) / height (cm)] x 10,000 With a BMI of 16, evaluation of eating DO, anemia, or other underlying medication problems needed 1) Assess daily nutrition and exercise, assess anorexia with or without binging and purging, labs work for CBC with different, electrolytes, thyroid, and urinalysis.

Lorazepam is the preferred BZD when managing alcohol withdrawal symptoms in a patient w/ comorbid?

Hepatic disease BZDs have long been used to treat withdrawal. Recommended guidelines use diazepoxide (librium) or diazepam (valium), agents w/ a long half-life in pts with adequate hepatic or live function. However, in pts with compromised hepatic function a bzd with shorter half-life such as lorazepam is recommended to reduce risk of toxicity and further liver damage

When conducting a neurological examination on a client, the PMHNP asks the client to hold out her arms and stick out her tongue while assessing for tremors. Which cranial nerve is being assessed?

Hypoglossal The tongue is controlled by hypoglossal cranial nerves.

Decreased levels of calcium in the bloodstream is indicative of what type of disease process?

In acute renal failure, calcium levels decrease. On the other hand, in disease processes such as acidosis, Addison's Disease, and hyperthyroidism, calcium levels are known to be increased.

Inferential statistics ?

Inferential statistics are numerical values that enable a researcher to draw conclusions that extend beyond the immediate data alone. Examples include t-test, analysis of variance, Pearson's r correlation, probability, and p-value. Descriptive statistics are used to describe the basic features of the data in a study. They are numerical values that summarize, organize, and describe observations. Examples include mean, standard deviation, and variance.

What test must the PMHNP complete after TCAs?

Liver function tests (LFTs) TCAs impact liver.

A 74-year-old married white woman was referred to you by her primary care provider for a psychiatric evaluation. She had a normal medical and neurological examination in the last 2 months. The client presents with her husband of 45 years who states, "My wife is just not the same anymore, she is irritable and asks the same question several times, even though I've answered it many times." The client responds, "Oh, Henry, you do the same thing, it's just a normal part of getting older, and the kids think everything is fine." During the assess- ment you compete the mini mental status examination (MMSE) and the client scores 18 as the PMHNP treating the client, you know the results of her MMSE indicate which level of cognitive impairment?

Moderate cognitive impairment Total score of MMSE 30 25-30 questionable significance 20-25 MILD 10-20 moderate 10 or lower SEVERE

Mandatory reporting for elderly

Not all states have mandatory statues for reporting elderly abuse. If the elder does not want to report abuse, and is competent the NP must be okay with this.

Primary prevention care practice are an essential aspect of the PMHNP role. Which of the following is the best example of a primary care strategy for community behavioral health?

Parenting skills classess for pregnant adolescents

Dialectical behavioral therapy (DBT) affirms dialectical thinking, which involves examining and discussing opposing ideas to find the truth. This philosophy is a supportive principle of DBT training. The central dialectical pattern emphasized in DBT involves the tension between:

Radical acceptance and change DBT emphasis acceptance of the current reality of what is and the ability to engage in personal change.

Serotonin is produced in which of the following locations:

Raphe Nuclei

The risk of being overly empathethic as a therapist is commonly characterized by which of the following responses by the client?

Re-enacted attachment w/ dependency and acting out. Dependency often forms as a result of a parent's genuine affection, extreme attachment, and overprotection, being overly empathetic as a therapist can can foster transference. This can be followed by acting-out-to push away from dependent attachment on the therapist

Validating is a communication technique used by the PMHNP when responding to a client about their intentions and treatment goals in the context of the client's noncommittal approach to treatment.

Recognizing, not validating, is a communication technique used by the PMHNP when responding to a client about their intentions and treatment goals in the context of the client's noncommittal approach to treatment.

Rett Syndrome

Rett syndrome's etiology is unknown. However, it is believed that children suffering from Rett syndrome have A known, progressive, and deteriorating course of illness after an initial period without apparent disability A potential metabolic disorder Genetic mutation(s)

Samantha is a 26-year-old partnered woman who works full time as a teacher. She is in a long-term relationship with Mary and they are getting along well, and doing well financially. They have two children, ages 2 and 6. Samantha is seeing the PMHNP to address her concerns that she is feeling down and sad for no reason and states, "I know my life is going well but I just don't feel happy. I have always worried a lot and have been sad most of my life." As a PMHNP trained in transactional analysis (TA), you understand that personality is multifaceted and wonder if which of the following is affecting her ability to experience happiness:

She likely had traumatic event in her childhood and her thoughts and feelings r/t to the event are locked together in her brain and cannot be accessed. According to TA, when a person is traumatized the thoughts and feelings get tied together and the process of therapy is to unlock the two.

Systemic effects of hypernatremia include

Systemic effects of hypernatremia include restlessness, thirst, and fever.

Medicaid vs Medicare

The Centers for Medicare and Medicaid Services (CMS) administer the two major public insurance programs, Medicaid and Medicare. Medicaid is funded by both federal and state tax dollars. In 2011, Medicaid provided health care for 60 million eligible persons, 85% of whom were children. Those eligible for Medicaid include the following: Low-income children Low-income pregnant women Elderly and disabled individuals who qualify for the Supplemental Security Income Program Medicare is funded by federal tax dollars. In 2011, it provided health care for 47 million eligible people. Those eligible for Medicare include the following: The elderly age 65 and older having worked 40 quarters and paid Medicare taxes Certain younger individuals with disabilities Individuals with end-stage renal disease Individuals in need of a kidney transplant Individuals receiving Social Security Disability and who have amyotrophic lateral sclerosis

Who takes fault if a collaborating agreement between NP and physician happens?

The NP, **regardless of collaborating psychiatrist, it is the PMHNPs who is responsible for the patients under their care in the court of law.** The NP is considered an autonomous provider.

Determining the levels of care for elderly client and caregiver/

The critical factor in determining whether care may be maintained in the home versus necessity for supervised care in a resident home or skilled nursing home is whether the caregiver's health and ability, willingness and motivation to continue to care for patient. Interventions will be contingent upon this. As a person's care requirement may increase over time, the complexity of the care needed can take a toll on the caregiver. Problems arise when the caregiver's health starts to suffer or the aging person needs exceeds the caregivers ability and resources

Who implements the CLozapine Risk Evaluation and Mitigation Strategy?

The manufacturers of clozapine

Alpha-agonists such as clonidine and guanfacine are often used to treat anxiety in children.

True Correct answer: True The FDA has approved certain medications for children in the treatment of anxiety. Due to the safe nature of these pharmaceuticals, they are appropriate for using off-label for anxiety.

Which moral duty does a nurse practitioner leader uphold when he or she tells the truth?

Veracity An important aspect of the nurse practitioner role encompasses the moral duties, obligations, and responsibility of being a leader in health care. Ethical principles that provide the foundation and direction for true leadership include the following: *Justice*: Doing what is fair *Veracity*: Telling the truth *Respect*: Treating everyone with equal dignity *Fidelity*: Being true and loyal


Conjuntos de estudio relacionados

Care of Patients with Common Environmental Emergencies

View Set

ATI capstone management pre-assessment

View Set

National Law Class Quizzes w/answers&topic

View Set

Abdomen - Appleton and Lange Questions, Abdomen - BRS Gross Anatomy 7th Ed, Abdomen-Big Picture Review Questions, Abdomen-Lippincott Review Question (Study w/Definition First)

View Set

Chapter 13, Chapter 14, Chapter 15

View Set

Guerrero - TExES 150 School Librarian Test Terms

View Set

chapter 3: neuroscience and behavior

View Set

Ch14 Commercial Vehicle Batteries

View Set

ECONMT - NEC 2014 Study Card Quiz

View Set

NSG 330 Ch 45- Management w/ Oral & Esophageal Disorders

View Set